Barkley PSYCH

अब Quizwiz के साथ अपने होमवर्क और परीक्षाओं को एस करें!

Which of the following would be considered a situational crisis? A. Diagnosed with a terminal disease B. Home burned down C. Caught in the midst of a riot D. Attacked by a thief

A person diagnosed with a terminal disease would be in a situational crisis because it is unplanned and arises from external events, such as loss of others or physical illness. A person having their home burned down, getting attacked by a thief, or getting caught in the middle of a riot would be experiencing an adventitious crisis because the events listed are unplanned, disastrous, and not typically considered everyday occurrences.

When you suspect delirium in a patient, which aspect of his or her health history does not indicate the disorder? A. Poor motor skills B. History of substance use and abuse C. Prior diagnosis of major neurocognitive disorder D. Nutritional status

Delirium impairs many mental capabilities such as memory and judgment, but motor skills would not be affected. A patient's nutritional status is relevant in providing information on a possible cause of delirium (e.g., vitamin B12 deficiency). Substance abuse, particularly ethanol abuse, is a known cause of delirium. Major neurocognitive disorder is a predisposing factor for delirium.

Kevin and Agatha's 5-year-old son, Ben, has been wetting the bed for the past 2 months. Kevin and Agatha are concerned that this could be a sign of other disorders that run in the family, as there is a history of diabetes and high blood pressure on Kevin's side. Which of the following conditions should be included in a differential diagnosis for Ben's condition? A. Kidney disease B. Diabetes mellitus C. Anxiety disorder D. High blood pressure

Diabetes mellitus should be suspected as a differential diagnosis in cases of enuresis, as the condition can present with nocturia, polyuria, and urgency. High blood pressure, kidney disease, and anxiety disorder are not considered as potential differential diagnoses for enuresis.

Part of your regimen for a group of patients with schizophrenia is to provide education on the disorder. You explain that there are many predisposing genetic, neurobiological, and environmental factors linked to schizophrenia. Regarding genetics, which of the following would be an accurate statement you could make to the group? A. Schizophrenia is a disorder that skips generations, giving children of persons with schizophrenia a lower risk than their grandchildren. B. Research has attributed chromosome 9 as the location. C. Risk is attributed to a spectrum of alleles, each of which results in minor, not complete, changes in expression. D. Children with one parent with schizophrenia have an 85% risk for the disorder.

Risk of schizophrenia is attributed to a spectrum of alleles; the genes result in minor, but not complete, changes in genetic expression or protein function. The condition has been associated with changes across multiple chromosomes, including chromosomes 6 and 22; however, chromosome 9 is not associated with the condition. Children with one parent with schizophrenia have a 13% risk, not an 85% risk, of developing schizophrenia. First-degree relatives of individuals with schizophrenia (e.g., children) have a much greater chance of developing the condition than second-degree relatives (e.g., grandchildren).

Which of these agents has been approved by the Food and Drug Administration for treatment of autism spectrum disorder? A. Methylphenidate B. Carbamazepine C. Risperidone D. Citalopram

Risperidone is approved by the Food and Drug Administration (FDA) for management of aggression in patients with autism spectrum disorder (ASD). Carbamazepine and other anticonvulsants do not play a role in the treatment of ASD. Fluoxetine and other SSRIs (e.g., citalopram) have shown some benefit in managing behavior in patients with ASD, and methylphenidate has been used to manage hyperactivity in patients with the condition; however, none of these agents have approved by the FDA for treatment of ASD.

Which of these agents is least likely to be used in treatment of alcohol withdrawal? A. Disulfiram B. Lorazepam C. Thiamine D. Selegiline

Selegiline, a monoamine oxidase inhibitor, is used in the treatment of Parkinson's disease, not alcohol withdrawal. Following the cessation of withdrawal signs and symptoms, patients may be given disulfiram to serve as a deterrent against future alcohol consumption. Lorazepam and other benzodiazepines are commonly used to manage the signs and symptoms of alcohol withdrawal. Thiamine is often administered in treatment to prevent possible cases of Wernicke encephalopathy.

During consultation with your patient, Shawna, it becomes clear that she has generalized anxiety disorder. Shawna asks you how common anxiety disorders are in the United States. Which of the following do you tell her? A. Generalized anxiety disorder is more common among young adults. B. Anxiety disorder is extremely common, with nearly 50% of the U.S. population developing an anxiety disorder in his or her lifetime. C. Approximately 2.9% of adults in the United States have generalized anxiety disorder in a given 12-month period. D. Generalized anxiety disorder is most common in African American individuals.

The 12-month prevalence of generalized anxiety disorder (GAD) is 2.9% among adults in the United States. The prevalence of GAD peaks in middle age and declines later in life; it is not more common among young adults. GAD is more common in individuals of European descent than individuals of other ethnicities, such as African Americans. Anxiety disorders are very common, with 12%-25% of the U.S. population experiencing pathological anxiety at some point in their lifetime, but not as many as 50% of the population.

During a routine examination for an inpatient, you find that he has contracted chlamydia. Which of these agents would you most likely prescribe him to treat the condition? A. Ofloxacin B. Levofloxacin C. Erythromycin D. Doxycycline

The Centers for Disease Control and Prevention recommends doxycycline and azithromycin as first-line agents for the treatment of chlamydia. Levofloxacin, ofloxacin, and erythromycin are all recommended as alternative treatments for when doxycycline and azithromycin are contraindicated.

Which of these portions of the brain is believed to be one of the most involved features in anxiety disorders? A. Limbic system B. Corpus callosum C. Raphe nuclei D. Locus coeruleus

The raphe nuclei and the amygdala are believed to be the most involved circuits of the brain in anxiety disorders. Although the locus coeruleus, limbic system, and corpus callosum are all involved in anxiety disorders to varying degrees, dysfunction in the pathways of the raphe nuclei and amygdala are seen as primary influences on anxiety disorders.

When was the role of the nurse practitioner developed and why? A. During the 1970s, as a result of physician shortages in oncology B. During the 1950s, as a result of physician shortages in gynecology C. During the 1960s, as a result of physician shortages in pediatrics D. During the 1980s, as a result of physician shortages in gerontology

The role of the nurse practitioner (NP) was developed in the early 1960s as a result of physician shortage in pediatrics. The growth of NP programs soon ensued, with distribution of NPs in various practice settings emphasizing ambulatory and outpatient care. Movement of NPs expanded to the inpatient setting as a result of managed care, hospital restructuring, and decreases in medical residency programs.

During a clinical neurological study, Lana sits on the cold examination table with electrodes affixed to her scalp. She is then asked to recall the face of a dearly departed loved one. When asked if she has successfully done so, she nods and wipes away a tear. Which part of her brain is she using? A. Frontal lobes B. Parietal lobes C. Temporal lobes D. Occipital lobes

The temporal lobes of the brain are involved in memory, emotions, and cognition. The occipital lobes are responsible for initially processing visual input, whereas the parietal lobes receive and process sensory input. The frontal lobes are responsible for voluntary motor and speaking activities, elaboration of thought, and executive functions.

You score an 82% on a test you had taken on two prior occasions. The last time you had taken the test, several months ago, you scored an 81%; the time before that, your score was 80%. Given these results, what does this test show in its ability to gauge your knowledge? A. Reliability B. Standard deviation C. Confidence interval D. T-test

Reliability, in research terms, is the consistency of a measurement, or the degree to which an instrument measures the same way over time with the same subjects; in this case, the participant's performance is similar over three tries, which shows that the test is consistent in its measurement of the participant's knowledge of the same subject. The confidence interval is an estimated range of values that covers the specified probability of including the parameter being estimated, with smaller intervals indicating a very precise range of values. Standard deviation indicates the average amount of deviation of values from the mean; 68% of all values fall within one standard deviation from the mean, while 95% of all values fall within two standard deviations. A t-test is a statistical test to evaluate the difference in means between two groups.

Michael, a 37-year-old male, has sought help for his use of alcohol. He says that he started with "one beer a night, but that worked its way up to three or four." He has attempted to cut back, but "it's been useless." He finds that he has a craving for alcohol when he has no access to it and that he has stopped going to social functions at bars or restaurants because he "makes a mess of things." Judging by these symptoms, how would you diagnose the severity of Michael's alcohol use disorder? A. Mild B. Severe C. Intermittent D. Moderate

A diagnosis of moderate alcohol use disorder requires four to five symptoms of the condition, which are demonstrated by the patient's increased consumption of alcohol over time, unsuccessful efforts to reduce consumption, cravings for alcohol, and continued use despite social or interpersonal problems. A diagnosis of mild alcohol use disorder would require two to three symptoms, and a diagnosis of severe alcohol use disorder would require six or more symptoms. There is no "intermittent" specifier for alcohol use disorder.

You are treating Pat for schizophrenia. She has begun developing signs of agranulocytosis and is bothered by significant weight gain. Based on these findings, you determine that you need to change her medication. Which of the following was she most likely prescribed originally? A. Fluphenazine B. Thiothixene C. Clozapine D. Haloperidol

Clozapine is strongly associated with a risk of agranulocytosis and. Like other second generation antipsychotics (SGAs), is closely associated with significant weight gain. First-generation antipsychotics (e.g. haloperidol, thiothixene, fluphenazine) may result in significant weight gain but are less likely to produce agranulocytosis than SGAs; in addition, haloperidol and fluphenazine are agents within this class associated with the least risk of weight gain and other metabolic side effects.

You are working with a university think tank on various health studies concerning the nine Southern California counties. In one study, the researchers have rated the counties from the healthiest to the least healthy. Using various designated health-related matrices, each county was given a ranking from 1 to 9. Which of the following is the last step the team takes in the research process? A. Analyzing the data B. Submitting for peer review C. Communicating the findings D. Interpreting the results

Communicating the findings is the final step in the 11 major steps in the research process for evidence-based practice and research methods. Analyzing data and interpreting the results are both steps leading to communicating the findings. Submitting for peer review is a general practice but not one of the major steps in the research process.

Which of the following statements is true about suicide? A. Hanging is the most common means of suicide for women. B. The majority of men tend to use guns to commit suicide. C. Overdose is the most common means of suicide for men. D. The majority of women cut veins or arteries to commit suicide.

In the general population, more than 60% of men use a firearm to commit suicide. More than 40% of women, not men, intentionally overdose on drugs to commit suicide. The most prevalent method of suicide in an inpatient psychiatric unit is hanging. Cutting a vein or artery is not the most common means of suicide in any gender population.

Which of these examples best fits Lawrence Kohlberg's interpersonal concordance orientation phase of moral development? A. A child knows that jaywalking is not necessarily wrong but waits for the turn signal to avoid punishment. B. A teenager does not cheat on his test because he knows it will disappoint his parents. C. An adult realizes that his community's rules on parking tickets effectively penalize the poor. D. A young adult does not shoplift from a store out of fear of being caught and prosecuted.

In Lawrence Kohlberg's model of moral development, the interpersonal concordance orientation phase is characterized by motivations based on the expectations of others (e.g., not wanting to cheat on a test because it will bring disappointment from one's parents). Individuals who know that there are different viewpoints on morality and punishment but still take actions out of self-interest (e.g., knowing that jaywalking is not necessarily wrong but waiting for the turn signal to avoid punishment) would be in the instrumental relativist orientation. The law and order orientation involves morality based on upholding the law of the society (e.g., avoiding shoplifting out of fear of being caught and prosecuted). The social contract legalistic orientation involves realizing that the rules of society are not in the best interests of individuals and may be harmful (e.g., realizing that a community's rules on parking tickets effectively penalize the poor).

Maggie has been your patient since she was born. She is now a 13-year-old adolescent. During a session with Maggie and her mother, you discuss the impending physical changes in Maggie's health. Maggie has begun her menstrual cycle and is interested in having a steady boyfriend. When should Maggie start receiving her Pap smear exam? A. About 3 years after the onset of vaginal intercourse but no later than 21 years of age B. At 21 years of age C. About 3 years after the onset of oral sex but no later than 21 years of age D. About 3 years after the onset of her menstrual cycle but no later than 21 years of age

It is recommended that female patients begin their annual Pap smear at 21 years of age. The American Academy of Family Physicians, the American College of Obstetricians and Gynecologists, and the Mayo Clinic do not recommend Pap smears for females younger than 21 years of age, regardless of sexual history, because most observed abnormalities in this population regress spontaneously. The onset of the menstrual cycle is not an indicator for when a female should begin her annual Pap smear. Oral sex is also not relevant, as the Pap smear examines the endocervical canal, not the mouth.

Nina is turning 40 years old next month and is saddened that she is still single and does not have a child. She and her therapist discuss her many past relationships and the reasons why none have lasted. Furthermore, they discuss her father leaving her family for another woman when Nina was 6 years old and how Nina has had trouble trusting men as a result. According to Harry Stack Sullivan's Interpersonal Theory, what major theme is presented in Nina's relationships? A. Grief B. Family modeling C. Role transitions D. Interpersonal conflicts

The patient's difficulty trusting men and sustaining long-term relationships represents Harry Stack Sullivan's theme of interpersonal conflicts, wherein difficult past relationships can lead to an inability to initiate or sustain close relationships. Family modeling is part of Eric Berne's model of transactional analysis, not Sullivan's model of interpersonal theory. Grief describes a bereavement following the death or loss of a loved one, which can then result in difficulty reestablishing interpersonal ties; however, the patient's difficulty in establishing relationships is based in abandonment, not loss. The theme of role transitions addresses problems with changes in life status and social or vocational roles (e.g., job loss, graduation from college).

Ernesto, a 32-year-old Hispanic male, reports to your clinic with findings that would suggest a panic attack, including trembling, a sense of discomfort in the chest, verbalized feelings of anxiety, and feeling "faint." When you ask him about his condition, he says he is having an ataque de nervios. What would be the best term to describe Ernesto's account of his condition? A. Cultural explanation B. Cultural idiom of distress C. Culture-bound syndrome D. Cultural syndrome

A cultural syndrome is when an individual of a culture experiences a condition with a specific cluster of signs and symptoms that is commonly recognized by the culture's society (e.g., ataque de nervios in a Hispanic patient); the culture may or may not recognize the condition as an illness, but an outsider aware of the pattern and its associated findings could recognize it as such. "Culture-bound syndrome" is an outdated term for cultural syndromes and similar culture-specific expressions of mental unease; it has fallen out of favor because it does not recognize experiences of distress and discounts the possibility of a wider distribution of cultural concepts of distress. A cultural idiom of distress is a term a culture uses to discuss a particular pattern of suffering or anxiety; unlike a cultural syndrome, there does not need to be a specific underlying condition or set of findings, and a generalized sense of discomfort may be the reason for the term (e.g. "feeling depressed" due to general ennui, rather than findings that suggest a depressive episode). A cultural explanation is when a culture creates a specific explanation for an illness or a cluster of findings (e.g., the "evil eye").

Chopra, who was born in India, is starting a new job at a local school and is required to take a TB screening test as a part of the hiring process. You injected purified protein derivative extract into Chopra's forearm. Three days later, you see a large swelling and redness at the injection site. Chopra says that he does not have TB but has always tested positive. To make sure, you order follow-up sputum cultures and chest X-rays for Chopra; neither of these exams show evidence of TB. What do the test results show? A. False positive B. True negative C. True positive D. False negative

A false positive presents when a diagnostic test shows a positive result for a disease the individual does not have. Many people born outside of the United States may have had a vaccine against TB, called bacille Calmette-Guerin, which can lead to a false positive on a purified protein test. A true positive occurs when the test is positive for a disease that the individual has. A true negative is a diagnostic exam that correctly identifies an individual as being free from the disease it assesses for. If the diagnostic showed the individual was healthy when he or she had the disease the diagnostic assesses for, that would be a false negative.

Which of the following provides a biological history that explores through generations? A. Vanderbilt Assessment Scale B. Patient Health Questionnaire C. Mini-Mental State Exam D. Family genogram

A family genogram will provide biological history and gather family process information, starting with the current family structure and working back through two generations. The Vanderbilt Assessment Scale evaluates the symptoms of a child with suspected attention-deficit/hyperactivity disorder. The Patient Health Questionnaire is a self-exam that screens for depression. The Mini-Mental State Exam is an examination that assesses for signs of cognitive disturbance.

Which of the following findings is more likely to be associated with anorexia nervosa than bulimia nervosa? A. Dehydration B. Esophagitis C. Lanugo D. Orthostatic hypotension

Lanugo, or fine, downy body hair, is more likely to produce in patients in anorexia nervosa. Orthostatic hypotension, esophagitis, and dehydration are all complications of repeat vomiting; although vomiting is commonly associated with bulimia nervosa, patients with anorexia nervosa may also exhibit purging behaviors.

You have been treating Antoine for his cocaine addiction. He comes to your office drunk and high on marijuana and says that you "can't say anything about it" because he didn't use cocaine. Antoine has used many other narcotics besides cocaine and has not followed the rehabilitation program you had outlined for him. Moreover, he has failed to pay for your services. Which of the following should you do? A. Call security. B. Give the patient notice and then dismiss him. C. Keep treating the patient, but sue him. D. Contact your supervisor.

A nurse practitioner (NP) has the right to dismiss a patient if he or she does not pay for services, persistently refuses to adhere to NPS recommendations, or is abusive. Calling security or contacting a supervisor are not steps in the process of dismissing a patient. The NP may continue to treat the patient for this session, then file a lawsuit after dismissing him, but suits against patients are not typically filed during continued treatment.

Antidepressants are drugs designed to reduce the symptoms of depression and are also used to treat anxiety and panic disorders. Which of the following is not a type of antidepressant? A. Monoamine oxidase inhibitors B. Lithium C. Selective-serotonin reuptake inhibitors D. Tricyclics

Lithium is prescribed in the treatment of patients with bipolar disorder, as opposed to depression. SSRIs, monoamine oxidase inhibitors, and tricyclics, on the other hand, are all types of antidepressants.

The mother of Jerome, a 9-year-old male, wants to discuss her son's recent behaviors. She states that when she asks him about possible involvement in extracurricular sports, Jerome answers, "What's the point? I'm not going to be good anyway." She has also noticed his lack of close friends, saying that he never wants to have other children over to the house. Basing your response on an understanding of Erik Erikson's psychosocial theory, you suggest Jerome's behavior is due to failure in which of the following conflict stages? A. Industry versus inferiority B. Intimacy versus isolation C. Identity versus role confusion D. Autonomy versus shame and doubt

According to Erik Erikson's psychosocial theory, industry versus inferiority occurs between the ages of 6 and 12 years and has the goal of self-confidence and peer recognition; failure at this stage can result in low self-esteem and poor interpersonal relations. Autonomy versus shame and doubt, which occurs between 18 months and 3 years of age, focuses primarily on confidence and willpower; those who succeed in this phase develop a sense of security and confidence, whereas those who fail may carry a sense of inadequacy and self-doubt. Intimacy versus isolation occurs in early adulthood, between the ages of 20 and 39 years; this stage is characterized by the patient learning how to open up to potential partners, with success leading to strong relationships and failure potentially leading to a lack of intimacy with partners. Identity versus role confusion, which characterizes the ages of 12-19 years, is defined by a struggle to identify oneself in relationship to one's peers and to develop a sense of personal identity and values.

John comes to your office complaining of pains in his abdomen after an appendectomy. An X- ray revealed some gauze in his abdomen that was accidentally left behind. Whom should you inform about this problem? A. The surgeon only B. John and the attending surgeon C. The clinic's lawyer D. The supervisor

According to the key ethical principle of veracity, which is the duty to be truthful, it is the nurse practitioner's duty to inform both the patient and the surgeon of medical supplies left in the surgery site. Informing the lawyer, supervisor, or only the surgeon, while failing to inform the patient, falls short of the aim of this ethical principle.

Adjustment disorders are emotional and behavioral reactions that develop within 3 months of a life stressor. These reactions are stronger than what one would expect for the event that occurred. There are many different stressors and reasons adults develop adjustment disorder. Which of the following is true in specifically diagnosing an adjustment disorder? A. The disturbance is merely an exacerbation of a preexisting anxiety disorder. B. The findings represent bereavement. C. Findings do not persist more than 6 months past the stressor's termination. D. The disturbance is indicative of another personality disorder.

Adjustment disorders are characterized by emotional or behavioral findings expressed in response to an identifiable stressor that occurred within the past 3 months, but findings typically do not persist more than 6 months past the termination of the stressor event. These findings are in excess of what would be expected, sometimes causing significant impairment in social or occupational (e.g. academic) functioning. Findings of adjustment disorders do not necessarily represent bereavement, as adjustment disorders may present as a result of many stressors. The disturbance is not necessarily indicative of another mental disorder (e.g., personality disorder) or an exacerbation of a preexisting mental disorder (e.g., anxiety).

All of the following patients have bipolar I disorder. Which of these patients is most likely be misdiagnosed with schizophrenia? A. Selena, a Mexican American woman B. Tanya, an African American woman C. Ken, a Japanese American man D. Connor, an Irish American man

African American patients with bipolar disorder are much more likely to be misdiagnosed with schizophrenia than Caucasian patients. A tendency towards misdiagnosis with schizophrenia has also been seen in Asian American and Hispanic American patients with bipolar disorder; however, this tendency is less commonly observed than misdiagnosis in African American patients.

Which of these patients would not meet one of the diagnostic criteria for gender dysphoria? A. Michael, an 8-year-old male who has preferred to play with girls for the past 2 years B. Tara, a 9-year-old female who has vocally expressed a desire for "boy parts" for 3 months C. Daniel, a 7-year-old male who has been playing with dolls for 1 year D. Alice, a 6-year-old female who has insisted on dressing in boys' clothes for over 8 months

Although a desire for biological features of one's experienced gender, rather than biological gender, is a diagnostic criterion for gender dysphoria, this finding would need to present for at least 6 months, not 3 months, to meet the criteria for the condition. Dressing in clothing of the other gender, seeking out playmates of the other gender, and engaging in play activities associated with the other gender are also possible criteria for gender dysphoria.

Which of the following is not part of the SIGECAPS mnemonic when analyzing a patient with possible depression? A. Anxiety B. Interest deficit C. Guilt D. Suicidality

Although anxiety may be comorbid with depression, it is not part of the SIGECAPS mnemonic used to assess patients for symptoms of major depressive disorder. Interest deficit, guilt, and suicidality, on the other hand, are all part of SIGECAPS (sleep, interest, guilt, energy, concentration, appetite, psychomotor, suicidal).

Which of these conditions is least likely to be considered a predisposing factor for violent behavior? A. Hypothyroidism B. Frontal lobe injury C. Tumors of the limbic system D. Low serotonin syndrome

Although hyperthyroidism may contribute to violent behavior, hypothyroidism is not commonly seen as a predisposing factor for the condition. Frontal lobe injury, as caused by blunt head trauma, is associated with outbursts of rage and violence. Low serotonin syndrome has been identified as contributing to aggression and violence, including violent suicidal behavior. Disorders and tumors of the limbic system have been associated with violent or aggressive behavior.

A 50-year-old father of twin high school graduates comes to your office concerned about the recent stress stemming from both of his daughters entering college. He also has a history of aggressive behavior linked with increased stress. Given this information, which of the following steps would be LEAST important prior to starting the interview? A. Arrange seating in your office so that a safe exit is possible, if necessary B. Have someone stand by for assistance in case the individual is a threat C. Consult with a member of the client's family concerning his or her impression of the problem D. Sit far enough away from the client for personal space

Although it is important to consult family members to get their impression of the presiding problem in the process of assisting a patient, the nurse practitioner (NP) should do so with the patient's permission, not prior to the interview. Proper seating arrangement, personal space, and having another person standing by for assistance are all steps that the NP should take to protect him or herself against an aggressive patient.

In the Diagnostic and Statistical Manual of Mental Disorders, 5th edition, which of the following is not a specifier for depressive disorders? A. With seasonal pattern B. With psychotic features C. With postpartum onset D. With atypical features

Although major depressive episodes (MDEs) may develop in the weeks or months following delivery, these conditions are specified as "with peripartum onset," not "with postpartum onset," as in 50% of all cases, these episodes first manifest during pregnancy. An MDE with atypical features characteristically presents with mood reactivity (i.e., mood brightening in response to positive events) as well as other distinctive features (e.g., hypersomnia, leaden paralysis). An MDE with psychotic features presents with hallucinations and/or delusions; the episode is further specified depending on whether these features are mood-congruent (i.e., characterized by guilt, death nihilism, deserved punishment) or mood-incongruent (i.e., presenting without the aforementioned themes or with a mixture of themes). Major depressive disorder (MDD) with seasonal pattern, or seasonal affective disorder, presents with a regular temporal onset of MDD associated with a particular time of year (e.g., winter), coupled with full remission or change to mania or hypomania at another characteristic time of year (e.g., spring).

Which of these assumptions regarding crisis intervention is correct? A. Crisis emerges as a result of a specific, identifiable event. B. The inclusion of family members in intervention will typically foment crisis and should be avoided. C. Crisis intervention involves probing the patient's history to find how it contributed to crisis. D. Crisis requires a prolonged intervention to resolve due to the many factors that provoked it.

Although many factors may lead to crisis, crisis emerges as a result of a specific, identifiable event that is often the target of crisis intervention. The inclusion of family members, significant others, or friends is not usually an added stressor that may be foment crisis; in many cases, the presence of loved ones and trusted associates can help to mediate the crisis. Crisis is an acute condition and is typically resolved within a brief time period; although some patients may require intervention afterwards to develop means of avoiding future crises, crisis intervention is not typically a prolonged process. Although crisis may emerge as a result of multiple factors, crisis intervention is directed solely towards resolving the hazardous event that provoked the crisis.

Which of the following most accurately defines anxiety? A. Intrusive, recurrent, persistent thoughts that interfere with activities of daily life B. Disturbance or alteration of integrative functions of identity, memory, or consciousness C. An insidious, diffuse, vague apprehension to an unknown threat that causes unpleasant feelings of unease or helplessness D. A sudden sense of dread and foreboding in response to a known, real, external threatening event

Anxiety is an insidious, diffuse, vague sense of apprehension that is often accompanied by autonomic symptoms. Anxiety produces an inappropriate or excessive response to a stimulus, resulting in feelings of impending danger caused by imbalances between the ego and external world or internal impulses. A sudden sense of dread and foreboding in response to a known, real, external threatening event describes fear rather than anxiety. A disturbance or alteration of integrative functions of identity, memory, or consciousness defines dissociative disorder. Intrusive, recurrent, persistent thoughts that interfere with activities of daily life characterize obsessive-compulsive disorder.

Hospitalization is one of a constellation of predisposing risk factors for delirium. Approximately 10%-15% of hospital patients, and up to 30%-40% of patients in intensive care or coronary care units, demonstrate symptoms of delirium. Leaving aside their hospitalization, which of the following patients carries the fewest predisposing factors for delirium? A. A 42-year-old with an active alcohol addiction recovering from knee surgery B. A 51-year-old admitted for dehydration as well as unrelieved pain which stems from bone cancer C. A 26-year-old blind patient admitted for withdrawal from benzodiazepine addiction D. An 88-year-old patient admitted with apraxia

As many as 80% of elderly inpatients experience delirium; however, apraxia alone is not a predisposing factor unless it is the result of Alzheimer's disease or some other dementia. All the other patients exhibit two risk factors for delirium, which include sensory deprivation (e.g., blindness), active substance use disorder, substance withdrawal, dehydration, pain, and periods of immobilization (e.g., recovery from knee surgery).

A 17-year-old presents to your office. She has lost 30 lb in the past year and weighs only 82 lb with a height of 65 in. Which of the following would be considered last as a nursing intervention when dealing with this patient? A. Improvement in body image and self-esteem B. Restoration of nutrition and hydration C. A strict exercise regimen to improve atrophied muscle D. Development of more adaptive coping mechanisms

As the patient's weight loss and low body mass indicate a likely case of anorexia nervosa, an exercise regimen to improve atrophied muscle should be considered only after the patient has recovered and weight has been safely restored. Restoration of nutrition and hydration, the development of more adaptive coping mechanisms, and the improvement in body image and self-esteem are all necessary components of nursing intervention for anorexia nervosa.

Which of these findings is not suggestive of cannabis withdrawal? A. Restlessness B. Increased appetite C. Depression D. Aggression

Decreased appetite, not increased appetite, is a diagnostic criterion for cannabis withdrawal. Aggression, restlessness, and depression are all criteria for cannabis withdrawal, as well as nervousness or anxiety, sleep difficulties (e.g., insomnia, disturbing dreams), and physical findings (e.g., abdominal pain, tremors, sweating, fever, chills, headache).

A patient presents to your clinic with complaints about difficulty losing weight. She is slightly overweight but complains that she feels "like a whale," saying that she has tried "everything" but still "balloons" regularly whenever she starts to lose weight. When you ask what weight loss measures she has taken, she is reluctant to answer. A physical examination reveals yellowed teeth, calluses on her knuckles, and petechial hemorrhaging. Given the patient's most likely condition, which of these findings would you least expect to find on a laboratory panel? A. Elevated blood urea nitrogen B. Low urinary pH C. Hypoglycemia D. Hyperkalemic metabolic acidosis

Bulimia nervosa is associated with patients with fluctuating weight who are intensely focused on their appearance and may present with yellowed teeth, calluses on the knuckles, and petechial hemorrhaging due to vomiting; although bulimia nervosa may present with hypokalemic metabolic alkalosis due to vomiting or normokalemic metabolic acidosis due to laxative use, hyperkalemic metabolic acidosis is not associated with the condition. In patients with bulimia nervosa, intravascular depletion may lead to elevated blood urea nitrogen, and dehydration may lead to hypoglycemia and low pH in urine.

You are counseling a patient who was referred to you after his second heroin overdose. He states he has his problem under control. You ask if he wants to continue putting himself in fatal situations and tell him that he should seek treatment because he is clearly not in control. Which of the counseling keystones are you applying in assessing the patient's crisis? A. Confrontation B. Concreteness C. Empathy D. Immediacy

By challenging the patient's behavior and statements, the counselor is applying the keystone of confrontation, where the analyst points out discrepancies between what the patient is saying and his or her actual behavior. Immediacy is the ability to focus on only the circumstances surrounding the hazardous event and striving for rapid solutions, realistic perceptions, and a decrease in tension. Empathy is the ability to understand and relate to another patient's emotional experiences. Concreteness involves having the patient specifically and accurately label his or her feelings and experiences.

Your client, David, tells you that he was so angry the other day that he yelled at his wife over recent purchases that cut deeply into their joint savings. You respond without judgment but with empathy, permissiveness, and unconditional positive regard toward your patient. You ask him, "How do you feel about that?" Of the following, from what psychotherapy theory are you most likely basing your practice? A. Person-Centered Therapy B. Gestalt Therapy C. Psychoanalytic Theory D. Behavioral Therapy

Carl Rogers' model of Person-Centered Therapy focuses on developing the client's sense of self-direction; in this model, the nurse practitioner (NP) expresses empathy, permissiveness, and unconditional regard towards the patient, asking without judgment how the patient feels about his or her actions. Behavioral Therapy stresses changing cognition, affect, and behavior by teaching the patient new coping skills, improving communication, and breaking maladaptive habits. Gestalt Therapy would emphasize that the patient's feelings and behaviors are ultimately his responsibility to address. Psychoanalytic Theory would focus on making the unconscious conscious and strengthening the ego so that the behavior is based more on reality and less on instinctual drives.

You suspect a patient in your care has bulimia nervosa. Which of these findings would be most suggestive of the condition? A. Loss of subcutaneous tissue B. Low body weight C. Pigmentation of the chest D. Dental caries

Dental caries are suggestive of the purging behaviors associated with bulimia nervosa. Pigmentation of the chest and loss of subcutaneous tissue are more likely to be seen in anorexia nervosa. Although patients with anorexia nervosa tend to exhibit a low body weight, patients with bulimia nervosa are more likely to exhibit normal body weight or be slightly overweight.

The parents of Justin, an 8-year-old male, come to you for help with their son's continuing behavioral problems. Twelve months ago, Justin was diagnosed with oppositional defiant disorder, and despite cognitive behavioral therapy for the last 8 months, Justin still has difficulty following direction, is defiant and moody, and lashes out frequently at his parents and teachers. Furthermore, Justin has few friends and has lost interest in most things he once enjoyed, including softball. His parents report that Justin will spend all day and night watching television if left to himself. You suspect that he may have a condition other than oppositional defiant disorder. Which of the following is the most likely diagnosis? A. Bipolar I disorder B. Bipolar II disorder C. Major depressive disorder D. Social anxiety disorder

Children with major depressive disorder (MDD) are often misdiagnosed with attention- deficit/hyperactivity disorder (ADHD) or oppositional defiant disorder; the patient's cluster of findings, which include irritable mood, markedly diminished interest or pleasure in activities, insomnia, and inability to concentrate, are all suggestive of MDD in children. Social anxiety disorder is characterized by intense fear in social situations, causing considerable distress and impaired ability to function in at least some parts of daily life. The patient's outbursts with his teachers and loss of friends do not appear to be caused by social anxiety. Bipolar I disorder is characterized by the occurrence of at least one manic or mixed episode, while bipolar II disorder is characterized by the occurrence of at least one hypomanic episode and one or more major depressive episodes; as the patient has not displayed any clear manic or hypomanic findings, neither condition would serve as a likely diagnosis.

Henry, a 28-year-old male, presents to your office with his wife, who has urged him to seek medical help for a 5-year history of what she describes as Henry's "mood swings." Henry says he experiences distinct periods of euphoria, during which he feels extremely competent, optimistic, and hyper-talkative. These episodes last for about a week and are usually followed by equally distinct periods of depression, during which he experiences fatigue, lack of motivation, and feelings of worthlessness. Henry has been able to function at work and maintain social relationships and responsibilities, although his friends have questioned his behavior at times. Henry had resigned himself to the fact that mild depression is in his nature, but he became alarmed after his depression grew more severe. Based on Henry's findings, which of the following is the most likely diagnosis? A. Major depression disorder B. Cyclothymic disorder C. Persistent depressive disorder D. Bipolar I disorder

Cyclothymic disorder presents with distinct episodes of hypomanic findings and depressive findings that last 2 years or more and do not meet the criteria for a manic or major depressive episode; during the course of the condition, findings are not absent for any period longer than 2 months. Individuals with cyclothymic disorder are often highly functional, and the disorder often goes unnoticed and undiagnosed. Bipolar I disorder is characterized by the occurrence of at least one manic or mixed episode. Major depressive disorder involves depressed mood or a loss of interest or pleasure in daily activities for at least a 2-week period. Persistent depressive disorder, formerly known as dysthymic disorder, is a mild, chronic form of depression; the condition does not present with hypomanic findings.

Kimberly has checked herself into your clinic following a suicide attempt. Which of the following is the first appropriate step in interviewing this patient? A. Assess the patient's protective factors. B. Assess the patient's risk factors to estimate risk. C. Practice active listening and establish rapport. D. Conduct an inquiry into suicide plans.

During an assessment interview with a suicidal patient, the nurse practitioner (NP) should begin with exploration, engaging in active listening and establishing rapport with the patient. In doing so, the NP prepares to assess risk factors, conduct an inquiry to assess the probability of an imminent suicide attempt (e.g., inquiring into how developed the patient's plans and thoughts regarding suicide are), and assess protective factors. The NP may perform these tasks by using a combination of a written self-report and a clinician-directed interview.

"Fight or flight" syndrome is the body's natural reaction to stress or perceived danger, and it prepares the body to fight or flee. Which of the following is not true regarding the body's "fight or flight" reactions? A. Heart rate increases, blood pressure increases; and gastrointestinal motility and secretions increase. B. Bladder smooth muscle contracts, bladder sphincter relaxes, secretion from the sweat glands increases. C. Hypothalamus stimulates sympathetic nervous system, sympathetic nervous system stimulates adrenal medulla, adrenal medulla releases epinephrine and norepinephrine. D. Pupils dilate, secretions from the lacrimal glands increase, respiration rate and depth increases.

During the "fight or flight" response, gastrointestinal motility and secretions decrease rather than increase; this occurs so the body can devote as much energy into the organs or secretions necessary to fight or flee. In the bladder, the smooth muscle contracts while the sphincter relaxes; this is seen as an evolutionary measure to promote voiding, so that the individual has less weight to load them down should flight become inevitable. The heart rate and blood pressure increase to ensure blood reaches all points of the body. The pupils dilate and lacrimal secretion increase to promote clarity of vision. Respiratory rate and depth increase to ensure more blood oxygen, while sweat secretion increases to promote cooling of the body's systems. The sympathetic nervous system is stimulated by the hypothalamus and adrenal medulla, with the latter releasing epinephrine and norepinephrine.

You are helping a patient to work through current complications in her life. At the start of the session, you explain to her the concepts of the parent, the adult, and the child, the three ego states that people have. Of the following, which therapy style are you practicing? A. Cognitive Behavioral Therapy B. Psychoanalytic Theory C. Transactional Analysis D. Person-Centered Therapy

Eric Berne's Transactional Analysis delineates the ego states into three concepts: The parent, the adult, and the child. Cognitive Behavioral Therapy focuses on collaborative empiricism, guided discovery, and cognitive processing of thoughts, emotions, and behaviors. The focus of Psychoanalytic Theory is to make the unconscious conscious, strengthening the ego so that the behavior is based more on reality and less on instinctual drives. Person-Centered Therapy emphasizes the therapist's qualities of empathy, permissiveness, and unconditional positive regard toward the client.

Travis is 22 years old and has been diagnosed with major depressive disorder (MDD). He has been receiving fluoxetine ever since his first episode; however, he recently experienced a relapse. Now that he is entering the maintenance phase of treatment, he wonders if he should continue the medication. He is discussing alternative options and even asks what the point of treatment is if it means he will relapse. Which of these statements might you make to him regarding treatment for MDD? A. "Electroconvulsive therapy is a good option for somebody in your position." B. "Amitriptyline is one of the first-line options for treatment of major depressive disorder." C. "If you take citalopram, you will never need to avoid red wine and other tyramine-containing foods." D. "Fifty percent of all patients who experience a major depressive episode will experience a relapse within 1 year."

Fifty percent of all patients who experience a major depressive episode (MDE) will experience a relapse within 1 year; 85% will experience a relapse within the course of their lifetimes. Amitriptyline and other tricyclic antidepressants were once commonly prescribed to manage major depressive disorder; however, as these agents produce side effects such as orthostatic hypotension, SSRIs are more likely to be prescribed as first- line measures these days. Monoamine oxidase inhibitors (e.g., selegiline) should not be taken alongside tyramine-containing food and drink (e.g., red wine); however, this is not a necessary precaution when taking citalopram, an SSRI. Electroconvulsive therapy is highly effective in managing MDEs; however, due to side effects such as postictal confusion and short-term memory difficulties, this intervention is typically reserved for MDEs refractory to antidepressants, presenting with psychotic features, or requiring rapid antidepressant response.

Which of the following conditions is least likely to produce findings that could be confused with a mood disorder? A. Tumor B. Syphilis C. Endocrine disorder D. GERD

GERD is not known to produce findings of depression. Dysregulation of the endocrine system, on the other hand, may present with various neuropsychiatric symptoms that do include depression. Pituitary tumors and tumors in various other locations in the central nervous system may cause depressive signs and symptoms and can be misdiagnosed as depression. Syphilis is called one of the "great imitators" of other diseases; left untreated, it can progress to neurosyphilis and affect the brain, causing depression as well as other neuropsychiatric findings.

You are treating a 34-year-old female who seems to have a morbid fear of obesity and has lost 18 lb in the last year. Her weight has decreased from 120 lb to 102 lb. Although you say that she is underweight, she says that she still perceives herself as fat. Given the most likely diagnosis, which of the following would be an expected complication of her condition? A. Hypothermia B. Dark-colored urine C. Hyperglycemia D. Clay-colored stool

Hypothermia is a complication of anorexia nervosa, a condition which is indicated by this patient's morbid fear of obesity, disturbance in self-evaluation of body image, and the loss of over 15% of her total body weight. Clay-colored stool and dark-colored urine are both expected findings of a patient with hepatitis B. Patients with anorexia nervosa may exhibit hypoglycemia, not hyperglycemia, due to a combination of dehydration and malnutrition.

A patient reports to your clinic with feelings of anxiety and depression. He says his depression has presented with a general sense of fatigue. In your office, the patient complains of cold, even though the thermometer is set at 68° F. You order a series of lab panels, suspecting primary hypothyroidism. Which of these sets of findings would best serve to confirm this condition? A. Low TSH, low T3, low T4, high FTI B. Low TSH, high T3, high T4, high free thyroid index (FTI) C. High TSH, low T3, low T4, low FTI D. High TSH, high T3, high T4, high FTI

Hypothyroidism can present with anxiety, depression, fatigue, and cold intolerance; in cases of primary hypothyroidism, rooted in the thyroid, a patient will classically exhibit high TSH, low T3, low T4, and low free thyroid index (FTI). Findings of low TSH, high T3, high T4, and high FTI would be suggestive of hyperthyroidism, which does not present with cold intolerance. In a patient with abnormal hypothalamic-pituitary function, hyperthyroidism might be suggested by high TSH, high T3, high T4, and high FTI. Secondary hypothyroidism, rooted in the hypothalamus or pituitary, may present with low TSH, low T3, and low T4; however, it also typically presents with low FTI, not high FTI.

Amelia is a 66-year-old patient who has had three consecutive normal results of the Pap smear test. Beyond the usual aches and pains of the aging process, she is otherwise healthy and has no history of abnormal cervical cells or cervical cancer. When should the nurse practitioner examine her with a follow-up Pap smear? A. 2 years from now B. Discontinue C. Next year D. 5 years from now

In patients with no history of abnormal cervical cells or cervical cancer, Pap smears should be discontinued between the ages of 65 and 70 years after three consecutive normal results. No further follow-up Pap smears are necessary after this point.

Sue is a patient with bipolar disorder who has recently been diagnosed with cancer, undergoing severe mood swings in the process. During a follow-up visit, several weeks after completing her treatment, she is depressed and tells you that she feels hopeless. Which is most important to understand about her ongoing intervention? A. The patient needs an emphasis on disease management for her cancer diagnosis. B. Suicidal thoughts are an impulsive response to the current problematic situation. C. Hopelessness is an imminent predictor of suicide in instances of chronic physical or mental illness. D. The patient requires an emphasis of illness management for her bipolar disorder.

In those with chronic physical or mental illness, lingering feelings of hopelessness are the top predictor of eventual suicide as well as a dynamic predictor for imminent attempted suicide. In cases where a condition such as cancer presents alongside bipolar disorder, the nurse practitioner (NP) must provide both continued disease and illness management for both conditions, rather than treating one as a complication of the other. Although suicidal thoughts may be a response to immediate problems, these thoughts can also be long-term considerations in relating to seemingly intractable problems; the NP must not assume that these thoughts are directly rooted in one source or the other.

Twenty-six year-old Kenneth presents to your clinic for STI screening. He is concerned that he may have contracted an STI after having unprotected sex with several partners since his release from prison 6 months ago. Kenneth has a tough, menacing demeanor. During your interview, you learn that he has been in and out of juvenile hall and prison since 16 years of age for battery, domestic abuse, drug possession with intent to distribute, armed robbery, and car theft. When you ask him if he has informed his sexual partners of his concerns about having an STI, he replies, in so many words, that he could not care less about his partners. Kenneth's behaviors and attitudes are indicative of a personality disorder. Which of the following personality disorders best characterizes Kenneth? A. Narcissistic personality disorder B. Antisocial personality disorder C. Schizoid personality disorder D. Schizotypal personality disorder

Individuals with antisocial personality disorder are often erratic and dramatic, lack respect for the rights of others, and exhibit underlying rage, hostility, tension, and irritability. Narcissistic personality disorder involves an overall lack of interest or empathy in others as well as grandiosity and manipulative behavior; however, the condition is not typically associated with violence. The characteristic signs of schizotypal personality disorder are odd beliefs or magical thinking influencing behavior (e.g., "special powers"), eccentric appearance or behavior, and odd thinking and speech. People with schizoid personality disorder are significantly socially withdrawn and introverted; they may appear eccentric, isolated, or lonely to others.

Amanda, a 21-year-old female, had complications during her delivery and has not been allowed to see her baby. According to the bonding and attachment theories of Marshall Klaus, John Kennell, and John Bowlby, intimate mother-infant contact within the first hours after birth is important for emotional bonding to the baby. This attachment is necessary for infants to survive and can be viewed as most similar to what other ethological theory? A. Sensitive periods of development B. Imprinting C. Attachment theory D. Adaptation theory

Konrad Lorenz's theory of imprinting concerns the critical period after birth, in which the infant establishes an attachment to the first object he or she sees, following it as a "maternal" object; this theory shares many key features with Marshall Klaus and John Kennell's bonding theory, as well as John Bowlby's attachment theory. Adaptation theory focuses on how individuals' behaviors change to fit their environments and interactions; this theory does not directly relate to the period of imprinting. Mary Ainsworth's attachment theory focuses on the bond that develops between child and caregiver; however, the theory focuses more on the types of attachment that develop as an infant or toddler, rather than the bonds that develop in the first hours after birth. Robert Hinde's sensitive periods of development theory concerns the same issue but focuses more on the development of skills, languages, and emotional attachments during key periods; although the period of imprinting is one of these sensitive periods, Hinde's theory is more focused on the wider scope of capabilities and deficiencies that may develop across several sensitive periods.

During a routine assessment for a patient with schizophrenia, you hear an unusual heart sound. The S3 sound is followed by a systolic murmur; you are best able to auscultate it at the apex, where it sounds high-pitched. Which of these conditions is the patient most likely experiencing? A. Aortic regurgitation B. Mitral regurgitation C. Aortic stenosis D. Mitral stenosis

Mitral regurgitation presents with an S3 sound followed by a systolic murmur; this murmur is best heard at the apex and is characterized as musical, blowing, or high- pitched. Mitral stenosis is also best auscultated at the apex but produces with an S1 murmur with a "crescendo" rumble. Aortic stenosis is systolic, producing with a "blowing," harsh murmur best heard at the second right intercostal space (ICS). Aortic regurgitation produces with a diastolic "blowing" murmur that is best auscultated at the third left ICS.

Patricia, a 12-year-old female, is helped with her quality of life for her bipolar disorder through assistance provided at her school and home. Her parents are also trained in improving their skills in dealing with her bipolar disorder. What treatment modality does this scenario describe? A. Clubhouse Model B. Psychosocial Rehabilitation Model C. Multisystem therapy D. Wellness Model

Multisystem therapy is a service for youths with serious mental illness that provides services at the school and home, helping parents and teachers to develop skills to aid children in situ. The Wellness Model refers to a holistic approach in which mind, body, and spirit are integrated. The goals include orienting toward optimal health and well- being in a purposeful manner and helping the patient live life more fully; however, the patient's spiritual needs are not addressed in this scenario. In the Psychosocial Rehabilitation Model, also known as the Clubhouse Model, participants engage in restorative activities focusing on their strengths and abilities rather than their illness.

You are assessing Adam, a 25-year-old male, who has presented to your clinic. He seems hypervigilant and has his fists clenched. Assessing for the potential of violence, you start a brief interview with Adam. Which of these statements from him would least suggest the potential for violence? A. "It's hard to keep up with my own thoughts some days ..." B. "Feels like clear skies going forward." C. "I'm the only authority in this room." D. "Why all the questions? Trying to get secrets out of me?"

Negative appraisal of impending events may be seen as suggestive of potential for violence, but a positive appraisal of impending events (e.g., "Feels like clear skies going forward") is not. Racing thoughts (e.g., "It's hard to keep up with my own thoughts some days...") and grandiosity (e.g., "I'm the only authority in this room") can suggest a manic episode, which can lead to aggression and violent behavior. Paranoia (e.g., "Why all the questions? Trying to get secrets out of me?") may suggest the potential for violence, especially when paired with hypervigilance.

Which of the following STIS is the leading cause of female infertility in the United States? A. HIV B. Gonorrhea C. HPV D. Herpes

Of the choices, gonorrhea is the most common cause of infertility among women in the United States. The infection may not produce recognizable symptoms until complications (e.g., pelvic inflammatory disease) have occurred. Herpes and HIV cause infertility at much lower rates than gonorrhea. HPV rarely causes infertility.

Which of these findings is not associated with oppositional defiant disorder? A. Deliberately annoying behavior B. Frequent loss of temper C. Blaming others for misbehavior D. Arson

Oppositional defiant disorder (ODD) is not associated with acts of harm or destruction, such as arson; these acts are more likely to present in a patient with conduct disorder. The diagnostic criteria for ODD include frequent loss of temper, deliberately annoying behavior, and blaming others for one's own misbehavior.

Michael, a 32-year-old male, reports to your clinic with signs and symptoms of a panic attack. You help him to address the attack; once it is over, you explain the condition to Michael and ask him to come back in 1 month for a follow up. At the follow up, which of these developments would most strongly suggest that Michael has panic disorder? A. The patient visited a psychiatrist and was diagnosed with social anxiety disorder. B. The patient has avoided the gym because he says exercise caused the first attack. C. The patient had another panic attack 2 weeks ago. D. The patient asks about possible "excitatory" side effects of a new medicine he is taking.

Panic disorder is suggested if, following an initial attack, the patient has experienced a maladaptive behavioral change related to avoiding future panic attacks (e.g., avoiding the gym out of the belief that exercise caused the first attack) for at least 1 month. Patients may exhibit concerns about medication side effects that may produce a panic attack; however, if the patient is still taking the medicine while inquiring after the side effects, that does not necessarily suggest a maladaptive pattern of avoidance. Panic attacks can present in other anxiety disorders, and it is important to discern whether the panic attack presented in a setting related to the specific anxiety disorder (e.g., a patient with social anxiety disorder experiencing a panic attack in a social setting). Repeat panic attacks are not necessarily suggestive of panic disorder; rather, panic disorder is suggested when the patient is persistently worried about future attacks or changes behavior in an attempt to avoid these attacks.

Cherrie, a 41-year-old female, was referred to your office by a primary care nurse practitioner for counseling after having a "nervous breakdown." During your interview, you note that Cherrie is dressed very seductively and speaks in a sexually suggestive manner, using colorful language and dramatic gestures. She seems intent on pleasing you, and, several times during the interview, she makes comments such as, "You're the expert; I could only hope to be as smart as you." She responds dramatically to your suggestions with statements like, "Wow, you're so right; I should've thought of that." Cherrie's behavior is most characteristic of which of the following personality disorders? A. Narcissistic personality disorder B. Histrionic personality disorder C. Obsessive-compulsive personality disorder D. Borderline personality disorder

People with histrionic personality disorder exhibit dramatic and flamboyant behaviors, have strong dependency needs, are seductive, can be sexually aggressive, and often have difficulty acknowledging emotional content. People with narcissistic personality disorder (NPD) exhibit characteristics of grandiosity and arrogance, tend to have an overall lack of interest or empathy in others, and are extremely sensitive to criticism. A patient with NPD is less likely to yield to another's authority and more likely to act as the true authority in any situation. People with obsessive-compulsive personality disorder (OCPD) tend to have a pervasive pattern of preoccupation with orderliness, perfectionism, and mental and interpersonal control at the expense of flexibility, openness, and efficiency. OCPD is not associated with emotional deficits or bizarre sexual behavior. People with borderline personality disorder (BPD) make frantic efforts to avoid real or imagined abandonment, have a pattern of unstable and intense interpersonal relationships, have a markedly and persistently unstable self-image or sense of self, and tend to be impulsive. Patients with BPD may idealize lovers or engage in unsafe sex but do not characteristically exhibit inappropriate sexually seductive behavior in social or professional settings.

Ernie, a 66-year-old male, is a first-time patient who comes to you for help with his depression. Two years ago, Ernie was fired from his longtime job at a chemical plant because he constantly accused his coworkers of verbally attacking him. He admits that he is disapproving of women and people in general; in fact, Ernie has never been married. He comments during your interview that people are "deceptive, greedy, and selfish by nature." You note that he has a pattern of projection and appears cold and detached. Ernie says that the only person he ever trusted was his son, but the two have not spoken in nearly 4 years. Apparently, this was over a dispute regarding the care of Ernie's grandson. Ernie says, "I'm not that torn up; if he can't trust me, he can go hang." Which of the following best explains Ernie's behavior? A. Schizoid personality disorder B. Borderline personality disorder C. Avoidant personality disorder D. Paranoid personality disorder

Paranoid personality disorder is characterized by a perception of attacks on one's character or reputation that are not evident, a tendency to hold grudges, a belief that others are threatening or deceptive by nature, the use of projection as a defense mechanism, repeated questioning of the loyalty of others, and an inability to express warmth or caring for others. Avoidant personality disorder is characterized by avoidance of social situations or relationships out of a fear of rejection, disapproval, or criticism, not out of distaste towards others. People with borderline personality disorder have extreme difficulty being alone and will form any kind of relationship to avoid it. People with schizoid personality disorder exhibit significant social withdrawal; tend towards introversion; appear eccentric, isolated or lonely to others; generally perform best in jobs requiring little contact with others.

All of these patients have bipolar I disorder. Which of these patients is least likely to have their condition develop into serious mental illness? A. Jacob, who was misdiagnosed with depression and was treated with antidepressants for years B. Laura, who exhibits severe manic episodes at time of diagnosis C. Angela, who still experiences manic symptoms while taking anticonvulsants D. Michael, who was first diagnosed with bipolar disorder at 18 years of age

Patients who are diagnosed with bipolar I disorder early in life are more likely to receive appropriate treatment and avoid a course of severity and relapse that may lead to serious mental illness. Patients who experience symptoms resistant to treatment are more likely to experience relapse. If a patient with bipolar I disorder receives a misdiagnosis of major depressive disorder, he or she might receive treatment with antidepressants, which might promote rapid cycling and exacerbate manic symptoms. High severity of symptoms at time of initial diagnosis is a strong predictor for risk of serious mental illness.

Which of the following symptoms of schizophrenia is the most likely reason for a patient not starting initial medical compliance? A. Alogia B. Anosognosia C. Avolition D. Anhedonia

Patients with anosognosia exhibit a lack of insight and cannot recognize or appreciate that they have a serious psychiatric illness that is clinically evident; this presents a tremendous challenge to starting or ensuring medical compliance and increases the risk of nonadherence to treatment. Alogia is the lessening of speech fluency and productivity, thought to reflect slowing or blocked thoughts; this finding is often manifested as short, empty replies to questions. Anhedonia, or social disinterest and lack of pleasure, is sometimes mistaken for avolition, but the two are in fact distinct characteristics and do not serve as strong impediments to care. Avolition is a psychological state, commonly seen in patients with schizophrenia, characterized by general lack of drive or motivation to pursue meaningful goals.

Which of the following is not a desired outcome in the ongoing care of a patient with bipolar disorder? A. Patient self-monitoring for signs of an impending episode B. Patients and their significant others understanding the nature of bipolar disorder C. Client execution of a psychiatric advance directive and a durable power of attorney for health care D. Gradually weaning the patient off medications

Patients with bipolar disorder generally require lifelong treatment, and many require medications for life; therefore, weaning a patient off medications is not a goal in treatment for bipolar disorder. The desired outcomes for ongoing care of patients with bipolar disorder include patient and family understanding of the disorder, client self- monitoring for signs of an impending episode, and client execution of a psychiatric advance directive and durable power of attorney for health care.

Janice, a 28-year-old female, has severe abandonment issues and cannot stand to be alone. She has problems with spending and uses shopping as an outlet to both relieve her anxiety and avoid being alone in her apartment. As a result, Janice is severely in debt. For the last year, she has also started cutting herself to relieve anxiety. Which of the following personality disorders, in which self-mutilation is especially prevalent, does Janice most likely have? A. Borderline personality disorder B. Histrionic personality disorder C. Schizotypal personality disorder D. Schizoid personality disorder

People with borderline personality disorder (BPD) make frantic efforts to avoid real or imagined abandonment, have patterns of unstable and intense interpersonal relationships, have a markedly and persistently unstable self-image or sense of self, tend to be impulsive, and may try to mediate emotions or seek aid by cutting and other self- harming behaviors. People with histrionic personality disorder exhibit dramatic and flamboyant behavior as well, but this behavior typically results in seduction and sexual aggressiveness. People with schizotypal personality disorder have odd beliefs or magical thinking that influence their behavior, are eccentric in appearance or behavior, and display odd thinking and speech; people with BPD do not experience such delusions. Instead of having unstable and intense relationships, people with schizoid personality disorder are significantly socially withdrawn, introverted, and may appear eccentric, isolated, or lonely to others.

Sharon, a 53-year-old female, has a high-stress job as a real estate agent and comes to your office with complaints of restlessness, difficulty concentrating and retaining information, chronic muscle tension, and frequent headaches. She admits to using alcohol and food frequently for relaxation and stress relief. This only occurs on the weekdays, not on the weekends. Which of Hildegard Peplau's four levels of anxiety best describes Sharon's situation? A. Severe anxiety B. Moderate anxiety C. Panic anxiety D. Mild anxiety

People with moderate anxiety experience levels of restlessness, difficulty concentrating, difficulty retaining information, selective inattention, and physical and somatic symptoms not found in those with mild anxiety; in this state, individuals may increase their use of defense mechanisms (e.g., alcohol, "stress eating") as a means of coping. Severe anxiety would also present with diminished or scattered concentration with diminished perception and worsened somatic symptoms; in this state, however, the patient's behavior is entirely turned towards stress relief, and the anxiety response may continue after the stressor has abated. Panic-level anxiety may present with suicidal ideation, an inability to understand simple directions, and a complete inability to communicate. Mild anxiety is associated with the tensions of day-to-day living; in this state, motivation, alertness, and learning are increased but not in a way that is typically perceived as problematic.

All of the following are predisposing factors associated with antisocial personality disorder except which one? A. Parental deprivation in childhood B. Criminal behavior C. Low socioeconomic status D. Overachievement and perfectionism

Perfectionism and overachievement are less frequently seen in individuals with antisocial personality disorder (ASPD) but more often in individuals with obsessive-compulsive personality disorder. Seventy-five percent of all people convicted of crimes have characteristics of ASPD. Lower socioeconomic levels and parental deprivation during the first 5 years of life are also common in those with ASPD.

Which of the following is not an adaptation process included in Jean Piaget's dynamics of development? A. Assimilation B. Preoperational C. Accommodation D. Equilibration

Preoperational is the second of the four stages of development in Jean Piaget's cognitive development theory; it is not included as a part of the dynamics of developmental aspects within his cognitive theory. Assimilation is the dynamic by which a child incorporates a new object or situation into a schema in order to understand it (e.g., if a cheetah has four legs, fur, and a tendency to meow, then the child understands that it is a cat). Accommodation is the dynamic by which a child realizes that new information does not fit an existing schema and tries to reconcile it with his or her worldview (e.g., the man wears a blue shirt, blue pants, and a shiny thing on his breast, but he's a gas meter reader, not a cop). Equilibration is the process by which the child works to reach accommodation after being introduced to the new object or situation.

You are asked by the emergency room personnel to evaluate a patient who displays cognitive impairment. Which of the following factors would most strongly indicate that the patient is experiencing delirium rather than neurocognitive disorder? A. Rapid onset of cognitive impairment B. History of methamphetamine use C. History of major depressive disorder D. Altered state of consciousness

Rapid onset of cognitive impairment most often serves to distinguish delirium from neurocognitive disorder (NCD), which typically has a gradual onset. An altered state of consciousness is a common feature of NCD and delirium and would not be used to distinguish the two diagnoses. Patients who develop major depressive disorder (MDD) late in life may be at increased risk for Alzheimer's disease. Although use of and withdrawal from methamphetamine is associated with the development of delirium, withdrawal from methamphetamine can produce mild NCD, and use of methamphetamine can promote the emergence of NCD due to HIV infection.

Jack's father died of lung cancer after many years of battling the disease. His father's death has added to his stress from going through a divorce with his high school sweetheart. What kind of crisis is Jack experiencing? A. Situational B. Both situational and adventitious C. Personal D. Adventitious

Situational crises involve the loss of others through death, change in employment or financial status, change in the status of a significant other, or physical illness. An adventitious crisis is associated with chance rather than being an integral part of a situation, such as in natural disasters or crimes of violence; although these crises may result in trauma, the events behind the crisis are not generally seen as "a part of life." There is no "personal crisis" listed in medical terminology.

Randy, a 15-year-old male, has been under your care for anxiety disorder. Based on what his parents have told you, you must look into Randy's suicide risk. Which factor should not be included in your assessment? A. History of juvenile diabetes B. Economic background of the patient's family C. Family history of suicide D. Signs of an impulsive, dramatic personality

Socioeconomic background is not considered a factor in the assessment of suicide risk. Family history, on the other hand, is considered a risk factor, as is chronic illness (e.g., diabetes). Psychoanalytic signs, such as impulsive, dramatic personality disorders should also be assessed.

Which of these scenarios would be fully compliant with the Culturally and Linguistically Appropriate Services Standards? A. A doctor relies on the wife of his Mexican patient for translation services. B. A 24-hour clinic has a Korean-language translator available from 9 a.m. to 6 p.m. every day. C. The clinic provides patients with both notices and verbal offers in many languages informing them of their right to translation services. D. A clinic employs a Japanese interpreter whose credentials have not been renewed for 5 years.

Standard 5 of the Culturally and Linguistically Appropriate Services (CLAS) Standards holds that clinics must provide to patients both notices and verbal offers in the their language of choice informing them of their right to receive language assistance services. Standard 4 of CLAS requires that language assistance services be provided to patients at no cost, at all points of contact, and in a timely manner at all hours of operation; therefore, a 24-hour clinic that only has a Korean-language translator available for a limited set of hours would not meet this standard. Standard 6 requires that medical facilities do not rely on friends or relatives of the patient (e.g., the patient's wife) for translation services. Likewise, Standard 6 of CLAS requires that clinics assure the competence of language assistance services; a translator with outdated credentials would not meet this standard.

Wing-Chow, a Chinese American male who has no family history of prostate cancer, has come in to your clinic for a thorough physical exam. According to the American Cancer Society's guidance, what is your advice to him about when and how often he should have prostate screenings? A. After age 45 years, biannually B. After age 60 years, annually C. After age 40 years, biannually D. After age 50 years, annually

The American Cancer Society recommends an annual prostate screening for all males over the age of 50 years who have no history of prostate cancer and exhibit no significant risk factors. Screening does not start at age 60 years. Prostate-specific antigen (PSA) testing is available as an option starting at age 40 years and is recommended for patients who may be at higher risk for prostate cancer (e.g., a patient with a first-degree relative who experienced prostate cancer, African American men). Although the rate of testing may vary based on the patient's PSA blood test, it is never done on a biannual basis.

Jonathan, a patient with schizophrenia, has been living on the streets for about 10 years. He has discontinued the use of his medication and has not been in touch with his family for as long as he has been out on the streets. By what treatment modality would he be best served? A. Fairweather Lodge Model B. Clubhouse Model C. Assertive Community Treatment Model D. Psychosocial Rehabilitation Model

The Assertive Community Treatment Model is organized with a core service team with fixed responsibility delegated to a primary provider who provides continuity across time and functional areas; a homeless patient with serious mental illness would be best served by this program, as it delivers assertive outreach and treatment to where the client lives. The Psychosocial Rehabilitation Model, also known as the Clubhouse Model, is unique in that it is not a clinical program and there are no therapists or psychiatrists on staff. All member participation in the Clubhouse Model is strictly on a voluntary basis; therefore, this program would not work as well with patients who are not likely to involve themselves in group activities. The Fairweather Lodge Model is a psychosocial rehabilitation program combining congregate living with collaborative employment; however, this would not best serve a patient who lives on the streets.

Which of the following is not a Diagnostic and Statistical Manual of Mental Disorders, 5th edition, specifier associated with the diagnosis of schizophrenia? A. With catatonia B. Paranoid C. First episode D. Multiple episodes currently in partial remission

The Diagnostic and Statistical Manual of Mental Disorders, 5th edition, no longer uses subtypes of schizophrenia such as paranoid, disorganized, and undifferentiated. The catatonia specifier remains, however. Course specifiers such as "first episode" and "multiple episodes, currently in partial remission," may be used after a 1-year duration of the disorder.

Rebecca is receiving treatment for her symptoms of severe and persistent mental illness. This program focuses on a wellness model, using a 12-week course of weekly sessions to instruct Rebecca and her family on the particulars of her condition. What treatment modality is her program? A. The Three R's Psychiatric Rehabilitation Program B. Multisystemic therapy C. Psychosocial Rehabilitation Model D. Clubhouse Model

The Three R's Psychiatric Rehabilitation Program utilizes the wellness model, using a 12- session program to help instruct patients and their family members on the particulars of managing the patient's illness. Multisystemic therapy is a home-based service for youths with serious mental illness that provides services at the school and home and develops parents' skills to help their children; unlike the Three R's program, however, multisystemic therapy is not necessarily limited to a specific course of 12 sessions. The Psychosocial Rehabilitation Model, also known as the Clubhouse Model, is not a clinical program and provides no specific instruction for patients or family members.

In an effort to pursue a mental and physical fitness regime, Val has signed up for courses in tai chi and qigong. He was puzzled at first by the slow body movements and wondered how they would help him achieve his goals. By the end of the classes, he was impressed by how clear his mind was and how much the exercise made him sweat. Which part of the neuroanatomy becomes more active during activities such as tai chi and qigong? A. Temporal lobes B. Hypothalamus C. Frontal lobes D. Basal nuclei

The basal nuclei coordinates slow, sustained movements while suppressing extraneous or unnecessary patterns of movement; thus, this part of the brain would be stimulated by the deliberate motions of tai chi and qigong. The frontal lobes process voluntary motor and speaking activity, elaborative thought, and executive functions. The temporal lobes are involved in emotions, memory, cognition, speech, and behavioral awareness. The hypothalamus regulates homeostatic functions such as temperature control, thirst, and hunger sensations, as well as urine output.

Kathy, a 48-year-old female, has been gambling regularly for the past 5 years. However, it is only within the last 3 years that she has felt the problem getting out of control. Within this period, she claims to have lost over $150,000. A well-respected and successful professional, she admits she has become so excellent at deceiving people that no one else seems to know about her issue. She desperately wants to stop gambling. What method of treatment would you recommend to begin with? A. An SSRI B. An intervention from her close family and friends C. Buspirone D. A 12-step self-help program

The best treatment for gambling disorder follows a 12-step self-help program. Psychotherapeutic approaches, specifically Cognitive-Behavioral Therapy and Behavioral Therapy, are also useful in treating the condition. An intervention would not be necessary because this patient already realizes she has a problem and is looking for a solution. Little is known regarding the effectiveness of medication in treating gambling disorders, although SSRIs, naltrexone, buspirone, and mood stabilizers have shown efficacy in some studies.

Robert was fired from his job today. He comes home and berates his wife, Susan, for not finishing the household chores and having his dinner ready. "You're completely useless," Robert yells. "I don't know why I married you!" Robert's statement is most indicative of which of the following Freudian concepts? A. Identification B. Compensation C. Rationalization D. Displacement

The defense mechanism of displacement works to protect the ego by transferring feelings from one target to another that is considered neutral or less threatening (e.g., a fired worker yelling at his wife when he is angry at himself or his employer). Compensation refers to attempts to cover up a real or perceived weakness by focusing on a known strength. Identification is an attempt to increase self-worth by acquiring characteristics or attributes of an admired individual. Rationalization is an attempt to make excuses or formulate logical reasons to justify unacceptable feelings or behaviors.

In 2016, the Centers for Disease Control reported a total of 9,272 tuberculosis cases in the United States, a rate of 2.9 cases per 100,000. This was a decline of 3.6% from 2015. What is shown by this data? A. Prevalence B. Infectivity C. Incidence D. Trend

The description of the data shows prevalence, which is the proportion of a population that is affected by a disease or disorder at a particular time. Incidence is the rate of new occurrences of a disease in a population, thus indicating risk in a population; the data here clearly show the number of cases of tuberculosis in 2016, not the rate at which these cases occurred. Trend refers to long-term movement or change in frequency, whether upward or downward; as this scenario captures a specific moment in time, trend does not apply. Infectivity refers to the proportion of individuals exposed to a disease who will contract it; the figures in this scenario reflect spread but not rate of exposure.

Your patient, a 57-year-old woman, has recently been abandoned by her husband, who left her for a 27-year-old woman. She tells you that she feels despair that she might never find love again. You decide to use cognitive behavioral therapy to ease the patient's distress. Which of these statements would best demonstrate the principle of the downward arrow? A. "If that's the case, then what would you do to find fulfillment?" B. "Who do you think is truly at fault for the end of the marriage?" C. "I mean, being single at 57 years old is clearly the worst tragedy in modern life." D. "Many people your age have managed to find love after divorce."

The downward arrow approach aims to correct cognitive distortions by working from the assumption that the distorted thought is true and that there are steps for the patient to take to remedy it if the circumstances perceived are in fact correct (e.g. "If that's the case, then what would you do to find fulfillment?"). Paradox or exaggeration aims to defuse cognitive distortions by heightening them to the point that the ridiculousness of the belief becomes apparent (e.g. "I mean, being single at 57 years old is clearly the worst tragedy in modern life"); this technique should not be used unless the clinician has a close relationship with the patient, as it might be taken as mockery. Reattribution centers on critically examining one's role, as well as the roles of others, in an outcome (e.g. "Who do you think is truly at fault for the end of the marriage?"). Decatastrophizing involves exploring a cognitive distortion to show that it is not as horrible as the patient believes it to be (e.g. "Many people your age have managed to find love after divorce").

Lisa is under great stress in her personal and professional life. She has noticed that, in conjunction with her increased stress, she is becoming sick more often than usual. Which of the following hypothalamic axes are most likely involved in this situation? A. Hypothalamus-pituitary-circadian axis B. Hypothalamus-pituitary-gonadal axis C. Hypothalamus-pituitary-thyroid axis D. Hypothalamus-pituitary-adrenal axis

The hypothalamus-pituitary-adrenal axis releases corticotropin in times of stress, which can suppress the immune system, thereby making a person more susceptible to infection and disease during times of stress. The activity of the hypothalamus-pituitary-gonadal axis releases gonadotropin releasing hormone and follicle stimulating hormone, both of which are linked to estrogen secretion in females and sperm production in males, respectively. The hypothalamus-pituitary-thyroid axis regulates the basal metabolic rate and regular neurologic function; although its activity can lead to stress, it is neither activated by stress nor connected to the immune system. There is no hypothalamus- pituitary-circadian axis.

A patient under your care underwent a procedure outside the standard plan of care. The patient is covered under Medicare, and the procedure does not fall under "incident-to" billing. The Physician Fee Schedule values the procedure at $100. In terms of restitution, how much would you expect to receive from Medicare for the procedure? A. $35 B. $17 C. $68 D. $85

The nurse practitioner (NP) would receive $68 from Medicare for a procedure valued at $100. Under Medicare, NPs are reimbursed for physician services at 85% of the value listed on the Physician Fee Schedule; 80% of this reimbursement is paid by Medicare, and the remaining 20% is covered by the patient. As such, the NP would receive $68 from Medicare and $17 from the patient.

A 22-year-old recent college graduate admits he has been sleeping more than usual for the past month, sometimes up to 14 hours a night. However, he still feels significantly fatigued throughout his normal waking hours. He has no romantic relationships in his life, and his appetite has decreased, resulting in a 10-lb weight loss over the past month. When asked about his weight loss, he says that sometimes making the decision on what to eat is difficult. He also recently quit his band, saying that he is no longer interested in drumming. Which of the following issues is this patient most likely experiencing? A. Bipolar disorder B. Major depressive disorder C. Cyclothymic disorder D. Generalized anxiety disorder

The patient is already displaying sufficient findings to meet the criteria for major depressive disorder (MDD), including anhedonia, inability to make decisions, weight loss, hypersomnia, and chronic fatigue. Major depressive episodes (MDEs) can be a part of bipolar disorder, but manic episodes also alternate between the depressive episodes. Although patients with generalized anxiety disorder may be easily fatigued and experience sleep disturbance, the condition is characterized by excessive anxiety or worry, which the patient does not display. Cyclothymic disorder is indicated by a period of at least 2 years where the patient experiences alternating hypomanic and depressive findings without ever meeting the full criteria for a hypomanic episode or MDE; as the patient's depression is relatively recent and meets the full clinical picture for an MDE, cyclothymic disorder is not an option.

All of the following patients have recently been arrested for petty theft. Which of them would best fit the profile of a person with kleptomania? A. A 42-year-old woman with several children who has recently lost several jobs and is in severe debt B. A 16-year-old male who has a mother with depression and steals DVDs from a store with several friends C. A 20-year old woman who steals clothes from a store through an organized system to avoid detection D. A 32-year-old female who has a father with obsessive-compulsive disorder and steals lipstick regularly from grocery stores

Two-thirds of people with kleptomania are female, and there is usually a family history of obsessive-compulsive disorder in first-degree relatives. Individuals with kleptomania do not steal out of necessity or anger and typically focus on things they can easily afford. Thefts are usually performed alone, and although individuals with the condition tend to avoid stealing when immediate arrest is likely, the act is not preplanned, and patients with this condition do not generally take chances of apprehension fully into account.

Oscar, an 85-year-old African American male, is brought to the acute care clinic by his grandson. The family is concerned about Oscar's unusual level of confusion, which is much worse at night. The grandson tells you that, for the last 2 nights, Oscar became agitated during the night and didn't seem to know where he was. His grandson is especially worried because he discovered Oscar urinating on the back porch at 3 a.m. When examining his medical history, you note that Oscar was seen in the emergency room 2 days prior and started taking amoxicillin for a UTI. Oscar's most likely condition is which of the following? A. NCD due to Alzheimer's disease B. Transient ischemic attack C. Neurocognitive disorder (NCD) due to Lewy body disease D. Delirium

The patient's nocturnal behavior may be attributed to sundowning, an increase in agitation and confusion that presents closer to nightfall; this state is associated with delirium, which may present as a result of UTI in older patients. Confusion can result from a transient ischemic attack (TIA), of which African American men have a greater risk. However, a TIA more closely resembles a stroke, lasting 1-2 hours and presenting with dizziness, lack of coordination and balance, clumsiness or trouble walking, and muscle weakness of the face, arm, or leg. Although sundowning can also present in neurocognitive disorders (NCDs), such as Lewy body disease or Alzheimer's disease, the relatively rapid onset and short duration of the confused state does not suggest an NCD, which typically has a gradual onset.

Which of these statements from a patient experiencing delusions might suggest delusional disorder, unspecified type? A. "She says she's my mother, but I know it's somebody else wearing her face." B. "There are bugs crawling within my veins." C. "I know he loves me; he just says he doesn't know me because he's shy." D. "I'm on the verge of discovering a way to pioneer wireless energy."

The unspecified type applies to cases of delusional disorder where a patient's delusions have predominant beliefs that fall outside of the conventional categories; Capgras syndrome, where a patient believes that a familiar individual has been replaced by an imposter, is one such example of an unspecified delusional disorder (e.g., "She says she's my mother, but I know it's somebody else wearing her face"). Delusions focused on bodily functions or sensations (e.g., "There are bugs crawling within my veins") characterize the somatic type of delusional disorder. Delusions centered on the belief that a specific person is in love with the individual (e.g., "I know he loves me; he just says he doesn't know me because he's shy") characterize the erotomaniac type of delusional disorder. In the grandiose type of delusional disorder, the delusions may present with the belief that the individual is on the verge of a great discovery (e.g., "I'm on the verge of discovering a way to pioneer wireless energy").

Robert is a 16-year-old high school student. He has spent more time in his principal's office than the other students, due to a lack of control over his impulses and behavior. He is an average student and plays on the school baseball team, yet he has trouble interacting with other students off the field and outside the classroom. Robert has been caught stealing and making mischief throughout the campus. Which of the following may be expected when analyzing Robert's central nervous system, if he has a condition that causes poor impulse control and aggression? A. Low serotonin turnover with decreased 5-hydroxyindoleacetic acid in cerebrospinal fluid (CSF 5-HIAA) B. Low serotonin turnover with increased CSF 5-HIAA C. High serotonin turnover with decreased CSF 5-HIAA D. High serotonin turnover with increased CSF 5-HIAA

There is evidence that abnormal serotonin metabolism is present among individuals with severe impulse control problems, particularly low serotonin turnover with decreased 5- hydroxyindoleacetic acid in cerebrospinal fluid (CSF 5-HIAA). High serotonin turnover with increased CSF 5-HIAA is associated with major depressive disorder. As serotonin turnover is associated with generation of CSF 5-HIAA, low serotonin turnover is not likely to present alongside increased CSF 5-HIAA, and high serotonin turnover is not likely to present alongside decreased CSF 5-HIAA.

Jack, a 37-year-old male, presents with a 6-month history of panic attack signs and symptoms, including palpitations, sweating, hyperactivity, and "feeling hot." He has been reluctant to seek help because he does not want to "be stuck taking pills for the rest of [his] life." You run labs to rule out other conditions, which reveal low TSH, elevated T3, elevated T4, and elevated antithyroid peroxidase antibody. An electrocardiogram reveals supraventricular tachycardia. What do these results most likely indicate? A. lodine deficiency B. Toxic adenoma C. Autoimmune thyroid disease D. Hashimoto's thyroiditis

This patient's low TSH, elevated T3, elevated T4, elevated antithyroid peroxidase antibody (anti-TPO), and tachycardia all indicate a differential diagnosis of autoimmune thyroid disease, which is backed up by the patient's findings of palpitations, sweating, hyperactivity, heat intolerance, and tachycardia. Iodine deficiency would present with similar levels of TSH, T3, and T4, but it would also present with normal levels of anti- TPO. Hashimoto's thyroiditis is an autoimmune form of hypothyroidism; although this condition would present with elevated anti-TPO, it typically presents with elevated TSH, low T3, and low T4. Toxic adenoma is a hyperthyroid condition that would present with similar levels of TSH, T3, and T4; however, this condition would not present with elevated anti-TPO.

The Diagnostic and Statistical Manual of Mental Disorders, 5th edition, diagnostic criteria for schizophrenia require the presence of two or more distinct findings. Which of the following is not among those findings? A. Grossly disorganized or catatonic behavior B. Delusions C. Disorganized speech D. Two distinct identities or personality states

Two or more distinct or split identities or personality states are not a finding of schizophrenia; however, this finding is a diagnostic criterion for dissociative identity disorder, formerly known as multiple personality disorder. Although it is a common misperception that those diagnosed with schizophrenia present with split identities or personality states, neither is among the criteria for a clinical diagnosis. The Diagnostic and Statistical Manual of Mental Disorders, 5th edition, diagnosis of schizophrenia requires the presence of two or more of the following: delusions, hallucinations, disorganized speech, grossly disorganized or catatonic behavior, and negative symptoms (e.g., affective flattening, alogia, or avolition).

Morton and Grey have been in a relationship for a long time, but their relationship is often strained. During one argument, Morton insults Grey, erupting into a long rant and critiquing her viciously in several areas. The two have talked about the incident, but Morton has never apologized for it. The next day, he buys her a new car. According to Freud's theory of ego- defense mechanisms, what behavior is Morton displaying? A. Displacement B. Suppression C. Undoing D. Introjection

Undoing, according to Sigmund Freud's ego defense mechanisms, is symbolically canceling out a previous action that is intolerable; in this case, the husband dealt with the painful fact of his violence towards his wife by buying her a car, a symbolic act meant to cancel out his violence. Displacement is a transfer of feelings from one target to another that is considered less threatening or neutral, which is not relevant in this scenario. Introjection is a defense mechanism where the individual internalizes the voices of others to act as they would; as it is unclear whether the wife would feel a new car is a suitable apology for being verbally assaulted, the mechanism does not apply. Suppression is a blocking of unpleasant feelings and experiences from awareness; however, the husband is fully aware of his behavior.

A 24-year-old woman presents to your office after having been involved in a serious road rage incident. She was involved in a minor "fender bender," then she hit the other driver's car with her fist before driving away. She claims not to have been under the influence of drugs or alcohol at the time. When you ask what other factors in her life could cause such unacceptable behavior, she explains that she actually has "been in a very good place." She admits that this type of rage has happened a number of times in the past few months, though, and she is not proud of it: "Before it happens, I feel wound up like a spring. Afterwards ... I just feel awful." Along with a psychotherapeutic approach, you decide to treat her condition pharmacologically. Which of the following medications would not be used in her treatment? A. Propranolol B. Carbamazepine C. Venlafaxine D. Lithium

Venlafaxine is not used to manage intermittent explosive disorder (IED), which often involves episodes in which the patient cannot resist aggressive impulses that result in serious attacks and/or destruction of property; these episodes are typically followed by feelings of guilt or remorse. Fluoxetine and other SSRIs are the first-line treatment for IED, but anticonvulsants (e.g., carbamazepine) and lithium may be used in refractory cases. Some patients with IED have shown a positive response to propranolol and other beta-blockers.

Your patient, Edward, had a violent outburst today. He got up abruptly and threw a vase across the room, smashing it. What is the best course of action to take with Edward in this situation? A. Speak only to give short directions. B. Listen to his ranting, offering a sympathetic ear. C. Remain available to his need to vent. D. Remove yourself and others from immediate danger.

When a patient who is violence-prone is in active crisis, the best course of action is for the nurse practitioner (NP) to remove himself or herself, as well as other vulnerable people, out of the patient's range in order to minimize immediate danger. Active listening takes place during the postcrisis depression phase. Remaining available and speaking only to give short directions is what the NP should do during the de-escalation phase.

When educating the family or caregiver of a patient whose ability to function is impaired because of the onset of cognitive changes, you must first distinguish the nature of the condition from among several possibilities. Which of the following factors most strongly indicates neurocognitive disorder rather than delirium? A. The patient cannot answer questions. B. The patient appears to experience hallucinations. C. The patient's mood and affect are labile. D. Cognitive impairment is slow to develop, and the patient appears alert.

When discussing or diagnosing neurocognitive disorder (NCD), a factor which often distinguishes NCD from delirium is that the cognitive impairment of NCD usually develops slowly over a period of months or years. The patient may appear alert at time of diagnosis, whereas patients with delirium are not alert. In cases of both delirium and NCD, hallucinations may be present, and mood and affect may be labile. A patient with delirium may be unable to answer questions, whereas a patient with NCD may try to answer with nonsensical answers in an unconcerned manner.

Which of these statements from a patient regarding a recent diagnosis would best reflect a collateral-oriented relational orientation? A. "I don't care what my wife says; my health is my own concern." B. "I'm going to need to talk to my family about all of this." C. "What do you think I should do?" D. "My grandmother might know a thing or two about this ..."

When it comes to relational orientation, a collateral-oriented person is likely to believe the best way to organize is to come together as a group to make decisions; therefore, a person with this orientation may consult the family as a unit on how to proceed following a recent diagnosis (e.g., "I'm going to need to talk to my family about all of this"). Individualistic-oriented individuals are likely to see themselves as masters of their own destiny; individuals may consult their immediate family, but decisions are also likely to be made by the individual (e.g., "I don't care what my wife says; my health is my own concern"). Lineal-oriented individuals believe there is a natural order of relations and are likely to defer to a designated authority within the group, be it an elder within the family (e.g., "My grandmother might know a thing or two about this ...") or an outside expert, such as a medical professional (e.g., "What do you think I should do?").

Which of the following is not a key ethical principle for nurse practitioners? A. Nonmaleficence B. Autonomy C. Compassion D. Beneficence

While compassion may fall within the aims of the ethical principles for nurse practitioners (NPs), which are designed to govern conduct and thereby protect the rights of an individual, it is not an ethical principle in itself. Ethical principles for NPs include nonmaleficence, the duty to do no harm; beneficence, the duty to prevent harm and promote good; and autonomy, the duty to respect an individual's thoughts and actions.

Which of the following is a false statement about violence? A. The rates of violent incidents are greater in men among psychiatric patients. B. Domestic violence is the single most common cause of injury to women aged 15-44 years. C. Violence is associated more with socio-cultural factors than with psychiatric illness. D. Homicide is the third leading cause of death for youths aged 15-19 years.

Within the psychiatric patient population, the rates of violent incidents are similar for both sexes. Violence is more often associated with sociocultural factors such as poverty, unemployment, and child abuse than with psychiatric illness. Homicide is the third leading cause of death among youths aged 15-19 years, and domestic violence is the single most common cause of injury to women aged 15-44 years.


संबंधित स्टडी सेट्स

Cold War Big Ugly Test - Ambrose Ch.4 - Ch.9

View Set

Expresiones idiomáticas (Prueba 2)

View Set

Pediatric COMPREHENSIVE Practice Exam

View Set

3. Physics Practice Questions- Chapter 24

View Set

Chapter 42: Drugs Used to Treat Glaucoma and Other Eye Disorders

View Set

ITIL 4 Practitioner: Monitoring and Event Management

View Set

Programming Fundamentals I (Chapter 3)

View Set